When to use the root test. Is this not a good situation to use it? The 2019 Stack Overflow...

What does "rabbited" mean/imply in this sentence?

Which Sci-Fi work first showed weapon of galactic-scale mass destruction?

A poker game description that does not feel gimmicky

Realistic Alternatives to Dust: What Else Could Feed a Plankton Bloom?

Is "plugging out" electronic devices an American expression?

What could be the right powersource for 15 seconds lifespan disposable giant chainsaw?

Why is the maximum length of OpenWrt’s root password 8 characters?

Why do some words that are not inflected have an umlaut?

How are circuits which use complex ICs normally simulated?

Is there a name of the flying bionic bird?

Unbreakable Formation vs. Cry of the Carnarium

On the insanity of kings as an argument against monarchy

Why is Grand Jury testimony secret?

What is the meaning of Triage in Cybersec world?

Return to UK after being refused

What do hard-Brexiteers want with respect to the Irish border?

Limit to 0 ambiguity

How come people say “Would of”?

Why is it "Tumoren" and not "Tumore"?

Adding labels to a table: columns and rows

Is this food a bread or a loaf?

When to use the root test. Is this not a good situation to use it?

How to make payment on the internet without leaving a money trail?

Where to refill my bottle in India?



When to use the root test. Is this not a good situation to use it?



The 2019 Stack Overflow Developer Survey Results Are InWhich test would be appropriate to use on this series to show convergence/divergence?Integral test vs root test vs ratio testHow to show convergence or divergence of a series when the ratio test is inconclusive?Root test with nested power function?Confused about using alternating test, ratio test, and root test (please help).Radius and interval of convergence of $sum_{n=1}^{infty}(-1)^nfrac{x^{2n}}{(2n)!}$ by root and ratio test are different?How would I use root/ratio test on $sum_{n=1}^inftyleft(frac{n}{n+1}right)^{n^2}$?How would I know when to use what test for convergence?convergence of a sum fails with root testIntuition for Root Test.












2












$begingroup$


I'm having trouble seeing when to use the root test. nth powers occur, but I think the ratio test is easier:



enter image description here



Here is the problem:



$$sum_{n=1}^{infty} frac{x^n}{n^44^n}$$



So the ratio test seems to work here, but can't the root test be used to? The problem is that the $n^4$ doesnt play well with the root test right?



Here is the beginning of my solution with the ratio test:



$$biggr lbrack frac{a_{n+1}}{a_n} biggr rbrack = biggr lbrack frac{x^{n+1}}{(n+1)^4 * 4^{n+1}} * frac{n^4*4^n}{x^n} biggr rbrack = biggr lbrack frac{x*n^4}{(n+1)^4 * 4} biggr rbrack = frac{x}{4}$$



So I don't think the explanation for when to use the root test is totally right right? I can't really use it here because the $n^4$ causes some problems with the root test right?










share|cite|improve this question









$endgroup$

















    2












    $begingroup$


    I'm having trouble seeing when to use the root test. nth powers occur, but I think the ratio test is easier:



    enter image description here



    Here is the problem:



    $$sum_{n=1}^{infty} frac{x^n}{n^44^n}$$



    So the ratio test seems to work here, but can't the root test be used to? The problem is that the $n^4$ doesnt play well with the root test right?



    Here is the beginning of my solution with the ratio test:



    $$biggr lbrack frac{a_{n+1}}{a_n} biggr rbrack = biggr lbrack frac{x^{n+1}}{(n+1)^4 * 4^{n+1}} * frac{n^4*4^n}{x^n} biggr rbrack = biggr lbrack frac{x*n^4}{(n+1)^4 * 4} biggr rbrack = frac{x}{4}$$



    So I don't think the explanation for when to use the root test is totally right right? I can't really use it here because the $n^4$ causes some problems with the root test right?










    share|cite|improve this question









    $endgroup$















      2












      2








      2





      $begingroup$


      I'm having trouble seeing when to use the root test. nth powers occur, but I think the ratio test is easier:



      enter image description here



      Here is the problem:



      $$sum_{n=1}^{infty} frac{x^n}{n^44^n}$$



      So the ratio test seems to work here, but can't the root test be used to? The problem is that the $n^4$ doesnt play well with the root test right?



      Here is the beginning of my solution with the ratio test:



      $$biggr lbrack frac{a_{n+1}}{a_n} biggr rbrack = biggr lbrack frac{x^{n+1}}{(n+1)^4 * 4^{n+1}} * frac{n^4*4^n}{x^n} biggr rbrack = biggr lbrack frac{x*n^4}{(n+1)^4 * 4} biggr rbrack = frac{x}{4}$$



      So I don't think the explanation for when to use the root test is totally right right? I can't really use it here because the $n^4$ causes some problems with the root test right?










      share|cite|improve this question









      $endgroup$




      I'm having trouble seeing when to use the root test. nth powers occur, but I think the ratio test is easier:



      enter image description here



      Here is the problem:



      $$sum_{n=1}^{infty} frac{x^n}{n^44^n}$$



      So the ratio test seems to work here, but can't the root test be used to? The problem is that the $n^4$ doesnt play well with the root test right?



      Here is the beginning of my solution with the ratio test:



      $$biggr lbrack frac{a_{n+1}}{a_n} biggr rbrack = biggr lbrack frac{x^{n+1}}{(n+1)^4 * 4^{n+1}} * frac{n^4*4^n}{x^n} biggr rbrack = biggr lbrack frac{x*n^4}{(n+1)^4 * 4} biggr rbrack = frac{x}{4}$$



      So I don't think the explanation for when to use the root test is totally right right? I can't really use it here because the $n^4$ causes some problems with the root test right?







      sequences-and-series






      share|cite|improve this question













      share|cite|improve this question











      share|cite|improve this question




      share|cite|improve this question










      asked 4 hours ago









      Jwan622Jwan622

      2,38011632




      2,38011632






















          2 Answers
          2






          active

          oldest

          votes


















          4












          $begingroup$

          It doesn't cause any problems, because $lim_{ntoinfty}sqrt[n]{n^4}=1.$ Actually, the root test is stronger than the ratio test. Sometimes the root test limit exists, but the ratio test limit does not. However, if they both exist, then they are equal. Which is why if one limit is $1$ you shouldn't try the other, even though the root test is stronger.






          share|cite|improve this answer









          $endgroup$





















            2












            $begingroup$

            When doing a root test,
            powers of $n$ can be ignored
            because,
            for any fixed $k$,



            $lim_{n to infty} (n^k)^{1/n}
            =1
            $
            .



            This is because
            $ (n^k)^{1/n}
            =n^{k/n}
            =e^{k ln(n)/n}
            $

            and
            $lim_{n to infty} frac{ln(n)}{n}
            =0$
            .



            An easy,
            but nonelementary proof of this is this:



            $begin{array}\
            ln(n)
            &=int_1^n dfrac{dt}{t}\
            &<int_1^n dfrac{dt}{t^{1/2}}\
            &=2t^{1/2}|_1^n\
            &lt 2sqrt{n}\
            text{so}\
            dfrac{ln(n)}{n}
            &<dfrac{2}{sqrt{n}}\
            end{array}
            $



            Therefore
            $ (n^k)^{1/n}
            =n^{k/n}
            =e^{k ln(n)/n}
            lt e^{2k/sqrt{n}}
            to 1
            $
            .






            share|cite|improve this answer









            $endgroup$














              Your Answer





              StackExchange.ifUsing("editor", function () {
              return StackExchange.using("mathjaxEditing", function () {
              StackExchange.MarkdownEditor.creationCallbacks.add(function (editor, postfix) {
              StackExchange.mathjaxEditing.prepareWmdForMathJax(editor, postfix, [["$", "$"], ["\\(","\\)"]]);
              });
              });
              }, "mathjax-editing");

              StackExchange.ready(function() {
              var channelOptions = {
              tags: "".split(" "),
              id: "69"
              };
              initTagRenderer("".split(" "), "".split(" "), channelOptions);

              StackExchange.using("externalEditor", function() {
              // Have to fire editor after snippets, if snippets enabled
              if (StackExchange.settings.snippets.snippetsEnabled) {
              StackExchange.using("snippets", function() {
              createEditor();
              });
              }
              else {
              createEditor();
              }
              });

              function createEditor() {
              StackExchange.prepareEditor({
              heartbeatType: 'answer',
              autoActivateHeartbeat: false,
              convertImagesToLinks: true,
              noModals: true,
              showLowRepImageUploadWarning: true,
              reputationToPostImages: 10,
              bindNavPrevention: true,
              postfix: "",
              imageUploader: {
              brandingHtml: "Powered by u003ca class="icon-imgur-white" href="https://imgur.com/"u003eu003c/au003e",
              contentPolicyHtml: "User contributions licensed under u003ca href="https://creativecommons.org/licenses/by-sa/3.0/"u003ecc by-sa 3.0 with attribution requiredu003c/au003e u003ca href="https://stackoverflow.com/legal/content-policy"u003e(content policy)u003c/au003e",
              allowUrls: true
              },
              noCode: true, onDemand: true,
              discardSelector: ".discard-answer"
              ,immediatelyShowMarkdownHelp:true
              });


              }
              });














              draft saved

              draft discarded


















              StackExchange.ready(
              function () {
              StackExchange.openid.initPostLogin('.new-post-login', 'https%3a%2f%2fmath.stackexchange.com%2fquestions%2f3181802%2fwhen-to-use-the-root-test-is-this-not-a-good-situation-to-use-it%23new-answer', 'question_page');
              }
              );

              Post as a guest















              Required, but never shown

























              2 Answers
              2






              active

              oldest

              votes








              2 Answers
              2






              active

              oldest

              votes









              active

              oldest

              votes






              active

              oldest

              votes









              4












              $begingroup$

              It doesn't cause any problems, because $lim_{ntoinfty}sqrt[n]{n^4}=1.$ Actually, the root test is stronger than the ratio test. Sometimes the root test limit exists, but the ratio test limit does not. However, if they both exist, then they are equal. Which is why if one limit is $1$ you shouldn't try the other, even though the root test is stronger.






              share|cite|improve this answer









              $endgroup$


















                4












                $begingroup$

                It doesn't cause any problems, because $lim_{ntoinfty}sqrt[n]{n^4}=1.$ Actually, the root test is stronger than the ratio test. Sometimes the root test limit exists, but the ratio test limit does not. However, if they both exist, then they are equal. Which is why if one limit is $1$ you shouldn't try the other, even though the root test is stronger.






                share|cite|improve this answer









                $endgroup$
















                  4












                  4








                  4





                  $begingroup$

                  It doesn't cause any problems, because $lim_{ntoinfty}sqrt[n]{n^4}=1.$ Actually, the root test is stronger than the ratio test. Sometimes the root test limit exists, but the ratio test limit does not. However, if they both exist, then they are equal. Which is why if one limit is $1$ you shouldn't try the other, even though the root test is stronger.






                  share|cite|improve this answer









                  $endgroup$



                  It doesn't cause any problems, because $lim_{ntoinfty}sqrt[n]{n^4}=1.$ Actually, the root test is stronger than the ratio test. Sometimes the root test limit exists, but the ratio test limit does not. However, if they both exist, then they are equal. Which is why if one limit is $1$ you shouldn't try the other, even though the root test is stronger.







                  share|cite|improve this answer












                  share|cite|improve this answer



                  share|cite|improve this answer










                  answered 3 hours ago









                  MelodyMelody

                  1,07412




                  1,07412























                      2












                      $begingroup$

                      When doing a root test,
                      powers of $n$ can be ignored
                      because,
                      for any fixed $k$,



                      $lim_{n to infty} (n^k)^{1/n}
                      =1
                      $
                      .



                      This is because
                      $ (n^k)^{1/n}
                      =n^{k/n}
                      =e^{k ln(n)/n}
                      $

                      and
                      $lim_{n to infty} frac{ln(n)}{n}
                      =0$
                      .



                      An easy,
                      but nonelementary proof of this is this:



                      $begin{array}\
                      ln(n)
                      &=int_1^n dfrac{dt}{t}\
                      &<int_1^n dfrac{dt}{t^{1/2}}\
                      &=2t^{1/2}|_1^n\
                      &lt 2sqrt{n}\
                      text{so}\
                      dfrac{ln(n)}{n}
                      &<dfrac{2}{sqrt{n}}\
                      end{array}
                      $



                      Therefore
                      $ (n^k)^{1/n}
                      =n^{k/n}
                      =e^{k ln(n)/n}
                      lt e^{2k/sqrt{n}}
                      to 1
                      $
                      .






                      share|cite|improve this answer









                      $endgroup$


















                        2












                        $begingroup$

                        When doing a root test,
                        powers of $n$ can be ignored
                        because,
                        for any fixed $k$,



                        $lim_{n to infty} (n^k)^{1/n}
                        =1
                        $
                        .



                        This is because
                        $ (n^k)^{1/n}
                        =n^{k/n}
                        =e^{k ln(n)/n}
                        $

                        and
                        $lim_{n to infty} frac{ln(n)}{n}
                        =0$
                        .



                        An easy,
                        but nonelementary proof of this is this:



                        $begin{array}\
                        ln(n)
                        &=int_1^n dfrac{dt}{t}\
                        &<int_1^n dfrac{dt}{t^{1/2}}\
                        &=2t^{1/2}|_1^n\
                        &lt 2sqrt{n}\
                        text{so}\
                        dfrac{ln(n)}{n}
                        &<dfrac{2}{sqrt{n}}\
                        end{array}
                        $



                        Therefore
                        $ (n^k)^{1/n}
                        =n^{k/n}
                        =e^{k ln(n)/n}
                        lt e^{2k/sqrt{n}}
                        to 1
                        $
                        .






                        share|cite|improve this answer









                        $endgroup$
















                          2












                          2








                          2





                          $begingroup$

                          When doing a root test,
                          powers of $n$ can be ignored
                          because,
                          for any fixed $k$,



                          $lim_{n to infty} (n^k)^{1/n}
                          =1
                          $
                          .



                          This is because
                          $ (n^k)^{1/n}
                          =n^{k/n}
                          =e^{k ln(n)/n}
                          $

                          and
                          $lim_{n to infty} frac{ln(n)}{n}
                          =0$
                          .



                          An easy,
                          but nonelementary proof of this is this:



                          $begin{array}\
                          ln(n)
                          &=int_1^n dfrac{dt}{t}\
                          &<int_1^n dfrac{dt}{t^{1/2}}\
                          &=2t^{1/2}|_1^n\
                          &lt 2sqrt{n}\
                          text{so}\
                          dfrac{ln(n)}{n}
                          &<dfrac{2}{sqrt{n}}\
                          end{array}
                          $



                          Therefore
                          $ (n^k)^{1/n}
                          =n^{k/n}
                          =e^{k ln(n)/n}
                          lt e^{2k/sqrt{n}}
                          to 1
                          $
                          .






                          share|cite|improve this answer









                          $endgroup$



                          When doing a root test,
                          powers of $n$ can be ignored
                          because,
                          for any fixed $k$,



                          $lim_{n to infty} (n^k)^{1/n}
                          =1
                          $
                          .



                          This is because
                          $ (n^k)^{1/n}
                          =n^{k/n}
                          =e^{k ln(n)/n}
                          $

                          and
                          $lim_{n to infty} frac{ln(n)}{n}
                          =0$
                          .



                          An easy,
                          but nonelementary proof of this is this:



                          $begin{array}\
                          ln(n)
                          &=int_1^n dfrac{dt}{t}\
                          &<int_1^n dfrac{dt}{t^{1/2}}\
                          &=2t^{1/2}|_1^n\
                          &lt 2sqrt{n}\
                          text{so}\
                          dfrac{ln(n)}{n}
                          &<dfrac{2}{sqrt{n}}\
                          end{array}
                          $



                          Therefore
                          $ (n^k)^{1/n}
                          =n^{k/n}
                          =e^{k ln(n)/n}
                          lt e^{2k/sqrt{n}}
                          to 1
                          $
                          .







                          share|cite|improve this answer












                          share|cite|improve this answer



                          share|cite|improve this answer










                          answered 3 hours ago









                          marty cohenmarty cohen

                          75.2k549130




                          75.2k549130






























                              draft saved

                              draft discarded




















































                              Thanks for contributing an answer to Mathematics Stack Exchange!


                              • Please be sure to answer the question. Provide details and share your research!

                              But avoid



                              • Asking for help, clarification, or responding to other answers.

                              • Making statements based on opinion; back them up with references or personal experience.


                              Use MathJax to format equations. MathJax reference.


                              To learn more, see our tips on writing great answers.




                              draft saved


                              draft discarded














                              StackExchange.ready(
                              function () {
                              StackExchange.openid.initPostLogin('.new-post-login', 'https%3a%2f%2fmath.stackexchange.com%2fquestions%2f3181802%2fwhen-to-use-the-root-test-is-this-not-a-good-situation-to-use-it%23new-answer', 'question_page');
                              }
                              );

                              Post as a guest















                              Required, but never shown





















































                              Required, but never shown














                              Required, but never shown












                              Required, but never shown







                              Required, but never shown

































                              Required, but never shown














                              Required, but never shown












                              Required, but never shown







                              Required, but never shown







                              Popular posts from this blog

                              Why do type traits not work with types in namespace scope?What are POD types in C++?Why can templates only be...

                              Will tsunami waves travel forever if there was no land?Why do tsunami waves begin with the water flowing away...

                              Should I use Docker or LXD?How to cache (more) data on SSD/RAM to avoid spin up?Unable to get Windows File...